Eigvector/value problem (need validation of my proof)

  • Thread starter Thread starter rock.freak667
  • Start date Start date
  • Tags Tags
    Proof
Click For Summary
The discussion centers on validating a proof regarding eigenvalues and eigenvectors of matrices A and B. It confirms that if λ is an eigenvalue of matrix A with eigenvector e, and μ is an eigenvalue of matrix B with the same eigenvector e, then λ + μ is indeed an eigenvalue of the matrix A + B with e as the corresponding eigenvector. Participants note a typographical error where 'x' was used instead of 'e', which could indicate a misunderstanding of the notation. Despite this, the correctness of the proof's logic is affirmed. The consensus is that the proof is fundamentally sound, with minor notational adjustments needed.
rock.freak667
Homework Helper
Messages
6,221
Reaction score
31
[SOLVED] eigvector/value problem (need validation of my proof)

Homework Statement


Show that if \lambda is an e.value of a square matrix A with e as a corresponding e.vector, and [ites]\mu[/itex] is an e.value of the square matrix B for which e is also a corresponding e.vector,the \lambda + \mu is an e.value of the matrix A+B with e as a corresponding e.vector


Homework Equations





The Attempt at a Solution



From the def'n of an e.vector

Ax= \lambda x

Ax+Bx= \lambda x + \mu x

(A+B)x= (\lambda +\mu)x

hence \lambda +\mu is an e.value of A+B
 
Physics news on Phys.org
I don't see anything wrong with your proof. Although since the problem defines e as the eigenvector, it'd probably be more prudent (albeit arbitrary) to use that instead of x.
 
foxjwill said:
I don't see anything wrong with your proof. ... it'd probably be more prudent (albeit arbitrary) to use that instead of x.
I would consider the fact he used x when he should have been using e an actual error in his proof. Yes, it might simply be a persistent typographical error (which is still an error!) -- but it might also be a symptom of a deeper misunderstanding of how to manipulate mathematical statements.

However, I do completely agreement with you that the original poster's proof idea is correct.
 
ah yeah, a typo, supposed to be e since that is the e.vector.
 
Ah, there we go. Mystery solved. :smile:
 
Question: A clock's minute hand has length 4 and its hour hand has length 3. What is the distance between the tips at the moment when it is increasing most rapidly?(Putnam Exam Question) Answer: Making assumption that both the hands moves at constant angular velocities, the answer is ## \sqrt{7} .## But don't you think this assumption is somewhat doubtful and wrong?

Similar threads

Replies
6
Views
2K
Replies
2
Views
2K
Replies
5
Views
2K
Replies
6
Views
2K
  • · Replies 6 ·
Replies
6
Views
2K
  • · Replies 10 ·
Replies
10
Views
2K
  • · Replies 4 ·
Replies
4
Views
1K
Replies
4
Views
2K
Replies
20
Views
2K
  • · Replies 4 ·
Replies
4
Views
3K